2
$\begingroup$

Is it possible for me to construct a gate that inverse everything ($|0\rangle \rightarrow -|0\rangle, |1\rangle \rightarrow -|1\rangle$, etc. basically like a $-I$ gate) from the basic $X, Y, Z, CX,...$ gates, for any number of qubits? How do I do so if it's possible?

Thank you!

$\endgroup$

2 Answers 2

8
$\begingroup$

As a general rule, you wouldn't bother constructing this: it is just a global phase that has no observable consequence.

If you really insist on doing this, introduce an ancilla qubit in the $|1\rangle$ state and apply a $Z$ gate to it.

PS "inverse identity gate" is a really bad name for it. The identity operation is its own inverse.

$\endgroup$
2
  • 2
    $\begingroup$ The answer to this question (quantumcomputing.stackexchange.com/questions/2477/…) also gives a construction that does not need ancilla. It's construction is $\hat P_h(\theta)=\hat U_1(\theta)\hat X\hat U_1(\theta)\hat X$, and in your case, replace $\theta$ with $\frac{\pi}{2}$ $\endgroup$ Nov 24, 2020 at 11:43
  • $\begingroup$ They seem to be using "inverse" in the sense of additive inverse, but there really isn't much meaning to the additive inverse in QM. "Negation gate" would probably be a better phrasing. $\endgroup$
    – Tyberius
    Nov 24, 2020 at 20:18
4
$\begingroup$

You might be interested in controlled version of $-I$. Despite the fact that you can neglect global phase in case of non-controlled gates, you cannot do so in case of controlled version.

The controled gate $-I$ is described by matrix \begin{pmatrix} 1 & 0 & 0 & 0\\ 0 & 1 & 0 & 0\\ 0 & 0 & -1 & 0\\ 0 & 0 & 0 & -1\\ \end{pmatrix}.

This gate set a phase to $\pi$ (note that $\mathrm{e}^{i\pi} = -1$) if control qubit is in state $|1\rangle$.

To implement the gate simply put $Z$ gate on first qubit (i.e. control qubit) and nothing (i.e. identity operator) on second qubit (i.e. target qubit). You can check that the matrix above is really equal to $Z \otimes I$ and hence the proposed construction really implements the requested gate.

$\endgroup$

Your Answer

By clicking “Post Your Answer”, you agree to our terms of service and acknowledge you have read our privacy policy.

Not the answer you're looking for? Browse other questions tagged or ask your own question.